Epsilon delta limit problem

In summary, the problem requires finding an open interval around x0 where the inequality |f(x) - L| < ε holds, and determining a value for δ that satisfies 0 < |x - x0| < δ. Given f(x) = x^2, L = 3, x0 = -2, and ε = 0.5, the interval was initially thought to be (√3.5, √4.5), but the book states it should be the negative of those values. The correct interval is found by solving for x values at the end points of the inequalities, giving x = ±√3.5 and x = ±√4.5. The values near -2
  • #1
miglo
98
0

Homework Statement


find an open interval about x0 on which the inequality lf(x)-Ll<[tex]\epsilon[/tex] holds. Then give a value for [tex]\delta[/tex]>0 such that for all x satisfying 0<lx-xol<[tex]\delta[/tex] the inequality lf(x)-Ll<[tex]\epsilon[/tex] holds
f(x)=x2
L=3
x0=-2
[tex]\epsilon[/tex]=0.5


Homework Equations





The Attempt at a Solution


0<lx+2l<[tex]\delta[/tex] [tex]\Rightarrow[/tex] lx2-4l<0.5
-0.5<x2-4<0.5
3.5<x2<4.5
[tex]\sqrt{}3.5[/tex]<x<[tex]\sqrt{}4.5[/tex]

so i thought my interval about x0 would be ([tex]\sqrt{}3.5[/tex],[tex]\sqrt{}4.5[/tex]) but my book says its the negative of both those numbers, so I am thinking its because x approaches -2, then my interval would be on the negative side of the x-axis?
also i don't know how my book got [tex]\delta[/tex]=[tex]\sqrt{}4.5[/tex]-2 as an answer
please help me with these limit problems, I am really confused
 
Physics news on Phys.org
  • #2
miglo said:

The Attempt at a Solution


0<lx+2l<[tex]\delta[/tex] [tex]\Rightarrow[/tex] lx2-4l<0.5
-0.5<x2-4<0.5
3.5<x2<4.5
[tex]\sqrt{}3.5[/tex]<x<[tex]\sqrt{}4.5[/tex]

so i thought my interval about x0 would be ([tex]\sqrt{}3.5[/tex],[tex]\sqrt{}4.5[/tex]) but my book says its the negative of both those numbers, so I am thinking its because x approaches -2, then my interval would be on the negative side of the x-axis?
also i don't know how my book got [tex]\delta[/tex]=[tex]\sqrt{}4.5[/tex]-2 as an answer
please help me with these limit problems, I am really confused

When you solve for x values at the end points of your inqualities where you have x2 = 3.5 and x2= 4.5, you should get two solutions:

[tex]x =\pm \sqrt{3.5}\hbox{ and }x=\pm\sqrt{4.5}[/tex]

The values near x = -2 are the negative ones. You should draw this parabola and draw the lines y = 3.5 and y = 4.5 on your graph. You will see that x values near +2 and near -2 give y values between 3.5 and 4.5.

What remains is for you to find the correct interval for x nearby -2 and figure out what δ works. Note that |x + 2| < δ describes a symmetric interval about -2 and your interval isn't symmetric.

Edit: Isn't your L a typo and should be 4?
 

1. What is the Epsilon Delta Limit Problem?

The Epsilon Delta Limit Problem is a concept in mathematics that is used to determine the limit of a function as its input approaches a specific value. It is a way to rigorously define and prove the behavior of a function at a certain point.

2. How is the Epsilon Delta Limit Problem solved?

The Epsilon Delta Limit Problem is solved by using the definition of a limit and manipulating it to find a suitable value for delta, which is then used to determine a range for epsilon. This process involves using algebraic techniques and logical reasoning.

3. Why is the Epsilon Delta Limit Problem important?

The Epsilon Delta Limit Problem is important because it allows us to precisely define and prove the behavior of a function at a certain point. This is crucial in many areas of mathematics, such as calculus and analysis, where the concept of a limit is fundamental.

4. What are the applications of the Epsilon Delta Limit Problem?

The Epsilon Delta Limit Problem has many applications in mathematics, physics, and engineering. It is used to prove the convergence of sequences and series, to determine continuity and differentiability of functions, and to solve optimization problems.

5. Are there any limitations to the Epsilon Delta Limit Problem?

One limitation of the Epsilon Delta Limit Problem is that it can only be used to determine limits for functions that are defined on a continuous domain. It also requires a good understanding of algebra and logic, which may make it challenging for some individuals to grasp.

Similar threads

  • Calculus and Beyond Homework Help
Replies
9
Views
2K
  • Calculus and Beyond Homework Help
Replies
8
Views
1K
  • Calculus and Beyond Homework Help
Replies
19
Views
1K
  • Calculus and Beyond Homework Help
Replies
5
Views
883
  • Calculus and Beyond Homework Help
Replies
3
Views
5K
  • Calculus and Beyond Homework Help
Replies
16
Views
1K
  • Calculus and Beyond Homework Help
Replies
12
Views
2K
  • Calculus and Beyond Homework Help
Replies
5
Views
4K
  • Calculus and Beyond Homework Help
Replies
12
Views
2K
  • Calculus and Beyond Homework Help
Replies
7
Views
2K
Back
Top